Bạn chưa đăng nhập. Vui lòng đăng nhập để hỏi bài

Những câu hỏi liên quan
Bùi Minh Anh
Xem chi tiết
Mai Thanh Tâm
Xem chi tiết
Dương Bảo Lưu
Xem chi tiết
Chu Mạnh Đức
Xem chi tiết
ak5i5
Xem chi tiết
pgdtknd
Xem chi tiết
Lê Nhật Khôi
1 tháng 3 2018 lúc 21:24

Đây

Ta có: \(3^{2n}+3^n+1\)

Vì n không chia hết cho 3 nên: n có dạng là \(3k+1\)

Thế vào: Ta có: \(3^{6k+2}+3^{3k+1}+1\)

\(=729^k\cdot9+27^k\cdot3+1\)

Mặt khác: \(729\equiv27\equiv1\)(mod 13)

Do đó: \(729^k\cdot9+27^k\cdot3+1\equiv1\cdot9+1\cdot3+1=13\)(mod 13)

Vậy .............

P/s: Xét luôn trường hợp \(n=3k+2\)với cách làm tương tự trên

Yến Nhi
Xem chi tiết
Phạm Ngọc Linh
14 tháng 7 2016 lúc 15:46

oho

Yến Nhi
Xem chi tiết
không nói hahahahahha
16 tháng 7 2016 lúc 11:03

không trả lời

Yến Nhi
Xem chi tiết
không nói hahahahahha
16 tháng 7 2016 lúc 11:03

không trả lời

Mavis Vermilion
5 tháng 11 2016 lúc 12:45

sao ko ai trả lời